Mike has 21 pounds of tomatoes and 9 pounds of mixed peppers from his garden that he will use to make salsa and tomato sauce. The graph represents the system of equations shown.

A system of equations. x plus 5 y equals 21. x plus y equals 9.

The variable x represents the number of batches of salsa that can be made and y represents the number of batches of tomato sauce that can be made.

Answers

Answer 1

Answer:

x= 6 , y = 3

Step-by-step explanation:

x + 5y = 21

i am using trail and error method and i got the answer easily

the logic behind this is simple

the first equation is x + 5y = 21 so we can say that y  will be 5 x 1 , 5 x 2 , 5 x  3 , 5 x 4

first i did 5 x 4 = 20

x + 5 x 4 = 21

x= 1 but the equation x + y = 1 + 4 = 5  which is not equal to 9

second i tried with 5 x 3 =15

and the equation will be

x + 5 x 3 = 21

6 + 15 = 21

this time the second equation became true

x + y = 9

6 + 3 = 9

the no of pounds of salsa = 6

no of pounds of tomato = 3


Related Questions

Write the quadratic equation whose roots are 3 and -3, and whose leading coefficient is 3.
(Use the letter x to represent the variable.)

Answers

The quadratic equation is  3x² - 27.

What is Quadratic Equation?

A quadratic equation is a second-order polynomial equation in a single variable x ax²+bx+c=0. with a ≠ 0 .

Given:

The roots are 3 and -3

We have

α= 3   and β=-3

So,

α+β = -b/a  and αβ = c/a

α+β = 0 =-b/a and  αβ= -9 = c/a

Comparing from above

a=1, b= 0 and c=-9

Also, the leading coefficient is 3.

Then the quadratic equation is,

= 3(ax² + bx + c)

= 3( x² +0x- 9)

Hence, the quadratic equation is 3x² - 27.

Learn more about this concept here:

https://brainly.com/question/13771721

#SPJ1

If point d is placed on AC how will the measure of DAB relate to the measure of CAB

Answers

Angle DAB will be equal to angle CAB

What is trigonometry?

Trigonometry is the branch of mathematics which set up a relationship between the sides and angle of the right-angle triangles.

AC is a straight line and forms one of the sides of the triangle ABC. If point D is placed on line AC, angle DAB would remain equal to angle CAB since they are on the same line the angle formed with line AB remains the same.

To know more about Trigonometry follow

https://brainly.com/question/24349828

#SPJ1

help please
Find the sum or type
"impossible"
[3 -8] + [4 -5 -6]

Answers

The answer is negative 12

Use the grouping method to factor x³ + x² + 3x+3.
A. (x²+1)(x+3)
B. x(x+3)(x + 1)
C. (x + 1)(x+3)
D. (x + 1)(x2+3)

Answers

Answer:

D. [tex](x + 1)(x^2 + 3)[/tex]

Step-by-step explanation:

Hello!

We can group the first two terms and the last two terms.

Factor by Grouping[tex]x^2 + x^2 + 3x + 3[/tex][tex]x^2(x + 1) + 3(x + 1)[/tex][tex](x^2 + 3)(x + 1)[/tex]

Factoring by grouping is the process of breaking down larger polynomials to smaller ones to factor. We can then combine like factors.

In the second step, we can see that we can rewrite [tex]x^3 + x^2[/tex] as [tex]x^2(x + 1)[/tex], as both the two terms share a common factor of [tex]x^2[/tex]. We can pull out [tex]x^2[/tex] from that expression. Similarly, [tex]3x[/tex] and [tex]3[/tex] share a common factor of [tex]3[/tex], so we can pull that out.

Which of the following statements is false?

Answers

In quadrilateral PQRS, ∠P and ∠S are opposite angles.

Can someone help me please and thxxx

Answers

Answer:

C

Step-by-step explanation:

cos 30=12/x

[tex]x = 8 \sqrt{3} [/tex]

cos60=y/x

[tex]y =4 \sqrt{3} [/tex]

Answer:

x = [tex]8\sqrt{3}[/tex]

y = [tex]4\sqrt{3}[/tex]

Step-by-step explanation:

• tan 60° = 12/ y         [tan θ = opposite/ adjacent]

y = 12/ tan 60°  

y = [tex]4\sqrt{3}[/tex]

• x² = y² + 12²             [Pythagoras's theorem]

x² = ( [tex]4\sqrt{3}[/tex] )² + 144

x = [tex]\sqrt{(4\sqrt{3})^{2} \space\ + \space\ 144}[/tex]

x = [tex]8\sqrt{3}[/tex]

HELP PLSSSSSS hdghgdhj

Answers

Answer:

A

Step-by-step explanation:

Use the Pythagorean Theorem: a^2 + b^2 = c^2

The cable will be the hypotenuse, so c = 11.3 m

One leg is the distance of the anchor, so a = 4.2 m

(4.2)^2 + b^2 = (11.3)^2

17.64 + b^2 = 127.69

Subtract 17.64 from both sides

b^2 = 110.05

Square root both sides

b = ~10.5 m

NEED HELP TODAY PLEASE solve 4(2/5x+7)=−8−4/5x show your work

Answers

Answer:

x = -15

Step-by-step explanation:

please help guys how do i order fractions for least to greatest and greatest to least itll nean so much to me for my finals tmrw

Answers

Answer:

If the denominators are equal for all of the fractions, you can simply look at the numerator and order them. If the denominators aren't equal, you'll have to find the lcm (lowest common multiple), for example, if we have the fractions 3/6 and 1/4, we would find the lcm of 6 and 4, which is 12 and multiply the numerators accordingly. So the fractions would then become 6/12 and 3/12, this makes it easier for you to figure out their order from least to greatest.

Goodluck for your finals :)

Probability and Two-Way Tables

Answers

Answer:

the answer are A and Y , B and X.

 A and X are two independent events  , Option A is the correct answer.

When is Probability ?

Probability is the likelihood of an event to happen

A two way table is given

The two events will be independent when

If A is the first event and B is the second event , then

P(A) = P(A|B)

Here the P(A) = 30/100 = 0.3

P(B) = 20/100 = 0.2

P(A|X) = 15/50 = 0.3

P(A) = P(A|X)

Therefore A and X are two independent events , Option A is the correct answer.

To know more about Probability

https://brainly.com/question/11234923

#SPJ1

PLEASE HELP!!!
Part A: Solve the equation 4(x + 5) = 9x + 4x − 34, and show your work. (5 points)

Part B: Use this list of properties to justify each step of your math work. Justifications may be used more than once if needed. (5 points)


Distributive Property
Subtraction Property of Equality
Combine Like Terms
Multiplication Property of Equality
Division Property of Equality
Given
Addition Property of Equality

Answers

Answer:

See below

Step-by-step explanation:

4(x + 5) = 9x + 4x − 34

4x + 20 = 9x + 4x − 34   [Distributive]

4x + 20 =  13x − 34         {Combine Like Terms]

4x - 13x + 20 = 13x - 13x − 34  [Subtractive:  -13x both sides]

-5x + 20 = - 34            [Combine Like Terms]

-5x + 20 - 20 = -34 - 20     [Subtractive:  - 20 both sides]

-5x  = -54               [Combine Like Terms]

x = -54/-5             [Division Property:  divide both sides by -5]

x =  - 10 4/5            

In a group of 20 pupils, 3 play the flute only.
6 play the piano only.
6 play both instruments.
A student is chosen at random.
What is the probability the student plays neither instrument?


Please answer in fractional form!!

Answers

Answer
5/20
0.25

Step 1
Add all pupils that play the listed instruments

6+6+3= 15

Step 2
Subtract 15 from total number of pupils

20-15= 5

Step 3
You now have a 5/20 chance of choosing a pupil that does not play a listed instrument.

Answer:

[tex]\dfrac{1}{4}[/tex]

Step-by-step explanation:

Given information:

Total number of pupils = 203 pupils play the flute6 pupils play the piano only6 play both instruments

We can assume that 3 pupils play the flute only as we have also been told that 6 pupils play both instruments.

To calculate the probability that a randomly chosen pupil plays neither instrument, first determine how many pupils do not play an instrument by subtracting the number of pupils who do play an instrument from the total number of pupils:

⇒ total number of pupils - pupils who play instruments

⇒ 20 - (3 + 6 + 6)

⇒ 20 - 15

⇒ 5

Therefore, 5 pupils do not play the piano and/or flute.

To calculate probability:

[tex]\sf Probability\:of\:an\:event\:occurring = \dfrac{Number\:of\:ways\:it\:can\:occur}{Total\:number\:of\:possible\:outcomes}[/tex]

Therefore:

[tex]\implies \textsf{Probability of a pupil playing neither instrument} = \dfrac{5}{20}=\dfrac{1}{4}[/tex]

Situation:
A 45 gram sample of a substance that's
used to sterilize surgical instruments has
a k-value of 0.15.
N = Noe kt
No initial mass (at time t = 0)
N= mass at time t
k= a positive constant that depends on
the substance itself and on the units
used to measure time
t-time, in days
Find the substance's half-life, in days.
Round your answer to the nearest tenth.
Enter the correct answer.
DONE
+?

Answers

The substance's half-life is 4.7 days if the 11-gram sample of a substance that’s used to treat thyroid disorders has a k- the value of 0.15

What is exponential decay?

During exponential decay, a quantity falls slowly at first before rapidly decreasing. The exponential decay formula is used to calculate population decline and can also be used to calculate half-life.

We have an exponential function:

[tex]\rm N = N_oe^{-kt}[/tex]

Plug N = N⁰/2

[tex]\rm N_o/2 = N_oe^{-kt}[/tex]

[tex]\rm \dfrac{1}{2} = e^{-0.15t}[/tex]

Solving for t:

t = 4.62≈ 4.7 days

Thus, the substance's half-life is 4.7 days if the 11-gram sample of a substance that’s used to treat thyroid disorders has a k- the value of 0.15

Learn more about exponential decay here:

brainly.com/question/14355665

#SPJ1

Justin and elena each launched a toy rocket into the air. the height of justin’s rocket is modeled by the equation h = –16t2 60t 2. elena launched his rocket from the same position, but with an initial velocity double that of justin’s. which equation best models the height of elena’s rocket? h(t) = at2 vt h0 h = –16t2 60t 4 h = –32t2 120t 4 h = –32t2 60t 2 h = –16t2 120t 2

Answers

I don’t know you just have to add and keep adding them

What is the sum? startfraction 2 over x squared endfraction startfraction 4 over x squared endfraction

Answers

Answer:

  6/x²

Step-by-step explanation:

Rational expressions are added the same way numerical fractions are added. Their sum is the quotient of the sum of numerators, and the common denominator.

__

fraction sum

The given fractions already have the same denominator, so we simply add their numerators.

  [tex]\dfrac{2}{x^2}+\dfrac{4}{x^2}=\dfrac{2+4}{x^2}=\boxed{\dfrac{6}{x^2}}[/tex]

Unit 2 Show Your Work, BYU Geometry Part 1

Answers

The true statements about ∠MJK and ∠MJL are

The sum of the two angles is 180 degreesJK and JL are opposite raysJK and JL form a straight line

How to determine the true statements?

The statement is given as:

∠MJK and ∠MJL are a linear pair of angles and the angles are also supplementary

As a general rule, linear pair angles are adjacent angles that add up to 180 degrees. This means that:

They are supplementary anglesThey have a sum of 180 degreesThe angles may or may not be congruentThey have opposite rays i.e. straight line

The above means that, the following options are true:

(e), (f) and (h)

Also, the converse of the statement is:

∠MJK and ∠MJL are supplementary angles and the angles are also  linear pair of angles

This is false because not all supplementary angles are  linear pair angles

Read more about linear pair angles at:

https://brainly.com/question/1511741

#SPJ1

A stretch by a factor of 2 for the exponential growth function f(x)= a(nine-fourths) superscript x occurs when a = . a stretch by a factor of eleven-thirds for the exponential decay function f(x)= a(three-fifths) superscript x occurs when a = . a shrink by a factor of one-third for the exponential growth function f(x)= a(7)x occurs when a = . a shrink by a factor of two-fifths for the exponential decay function f(x)= a(two-ninths) superscript x occurs when a = .

Answers

Answer: A stretch by a factor of 2 for the exponential growth function f(x)= a(9/4)^x occurs when a =✔ 2

A stretch by a factor of 11/3  for the exponential decay function f(x)= a(3/5)^x occurs when a =✔ 11/3

A shrink by a factor of 1/3 for the exponential growth function f(x)= a(7)^x occurs when a =✔ 1/3

A shrink by a factor of 2/5  for the exponential decay function f(x)= a(2/9)^x occurs when a =✔ 2/5

Step-by-step explanation:

The exponential growth equation is stretched and compressed by a scale factor k = 2 , 11/3 , 1/3 and 2/5

What is exponential growth factor?

The exponential growth or decay formula is given by

x ( t ) = x₀ × ( 1 + r )ⁿ

x ( t ) is the value at time t

x₀ is the initial value at time t = 0.

r is the growth rate when r>0 or decay rate when r<0, in percent

t is the time in discrete intervals and selected time units

Given data ,

Let the exponential growth equation be represented as A

Now , the value of A is

For the exponential growth function f(x)= a(9/4)ˣ, a stretch by a factor of 2 occurs when a = 2

For the exponential decay function f(x)= a(3/5)ˣ, a stretch by a factor of 11/3 occurs when a = 11/3

For the exponential growth function f(x)= a(7)ˣ, a shrink by a factor of 1/3 occurs when a = 1/3

For the exponential decay function f(x)= a(2/9)ˣ, a shrink by a factor of 2/5 occurs when a = 2/5

Hence , the exponential equation is solved

To learn more about exponential growth factor click :

https://brainly.com/question/13674608

#SPJ5

Help needed
a hunid points :)

Answers

Answer:

the answer is C, 5a^6b

Step-by-step explanation:

The lengths represented by BE, EC, and CD on the diagram were determined to be 1800 feet, 200 feet, and 500 feet,
What is the length, in feet, across the lake?
Fill in the blank by entering only a number as your answer.

Answers

The length of lake is 4500 feet.

What is Triangle?

A triangle is a closed shape with 3 angles, 3 sides, and 3 vertices. A triangle with three vertices P, Q, and R is represented as △PQR.

Here, In ΔABE and ΔDCE

          ∠ABE = ∠DCE         { each 90⁰ }

          ∠AEB = ∠CED         { vertically opposite angle}

By AA similarity, we get

          ΔABE ≈ ΔDCE

Now, AB/CD = BE/CE

       AB/500 = 1800/200

       AB/500 = 9

        AB = 9 X 500

       AB = 4500 feet.

Thus, the length of lake is 4500 feet.

Learn more about Triangle from:

https://brainly.com/question/2773823

#SPJ1

What is the polynomial function of lowest degree with lead coefficient 1 and roots i, –2, and 2?

Answers

Answer:

x^4 - 3x^2 - 4.

Step-by-step explanation:

Imaginary roots  occur as conjugate pairs  so a 4th roots is -i.

So in factor form we have:

f(x) = (x - 2)(x + 2)(x - i)x + i)

     = (x^2 + 1)(x - 2)(x + 2)

     = (x^2 + 1)(x^2 - 4)

      = x^4 - 3x^2 - 4.

Answer:b.f(x) = x^4 – 3x^2 – 4

Step-by-step explanation:

If M is the midpoint in the figure, find MQ and PQ.

Answers

Answer:

pm = 4

M is the mid point of PQ which means that pm = mq

so MQ = 4

PQ = PM + MQ

= 4 + 4

= 8

Answer: A) MQ = 4, PQ = 8

Step-by-Step Solution:

Given :-

PM = MQ
PM = 4
M is the Midpoint of PQ

To Find :-

MQ and PQ

Solution :-

Since PM = MQ,
=> MQ = 4 (Since PM = 4)

PM + MQ = PQ (Since M is the Midpoint)
Hence, PQ = 4 + 4
=> PQ = 8

So, MQ = 4 and PQ = 8

A sign on a roadway at the top of a mountain indicates that for the next 4 miles, the grade is 10.5°. Find the change in elevation over that distance for a car descending the mountain. Round to the nearest hundredth.

Answers

the change in elevation is -0.73 miles (the negative sign is because the new elevation is smaller than the initial one).

How to find the change in elevation?

To do this, we can think on the situation as a right triangle, where the hypotenuse is 4 miles, the angle that we look at measures 10.5°, and the change in elevation (let's call it x) will be the opposite cathetus to that angle.

Then we can use the relation:

Sin(a) = (opposite cathetus)/(hypotenuse)

Replacing what we know, we get:

sin(10.5°) = x/4mi

x = sin(10.5°)*4mi = 0.73mi

So the change in elevation is -0.73 miles (the negative sign is because the new elevation is smaller than the initial one).

If you want to learn more about right triangles:

https://brainly.com/question/2217700

#SPJ1

1 − 2 + 3 −... + 99
Please solve!

Answers

Answer:

50

Step-by-step explanation:

So you can think of this by grouping it like this:

(1-2) + (3-4) + (5-6) + ... + (97-98) + 99

which is equal to: (-1) + (-1) + (-1)... + (-1) + 99

(each group is equal to -1, and 99 won't have a pair since it's the last one)

then, find how many groups of -1 there are:

the groups start at 1 and end at 98, but there are two in each group, so 98/2 = 49. this means there are 49 groups.

so now, you know that there are 49 -1s, so 49 * (-1) = -49.

finally, you can't forget the extra 99 that didn't have a pair, so -49 + 99 = 50.

a-3/7 devided by 3-a/21

Answers

The answer is pretty simple but I put it in a picture below

the bearing of two points x and y from z are 45° and 135° respectively . if |zx|=8cm and |zy|=6cm, find |xy|.

Answers

Answer:

[tex]|{\sf XY}| = 10\; {\rm cm}[/tex].

Step-by-step explanation:

Refer to the diagram attached. The dashed segment attached to [tex]\!{\sf Z}[/tex] points to the north. Rotating this segment clockwise with point [tex]{\sf Z}\!\![/tex] as the fixed center of rotation would eventually align this segment with the one between point [tex]\!\!{\sf Z}[/tex] and point [tex]\!\!{\sf X}[/tex]. The bearing of point [tex]{\sf X}[/tex] from point [tex]{\sf Z}[/tex] is the size of the angle between these two line segments when measured in the clockwise direction.

Subtract the bearing of [tex]{\sf Y}[/tex] from [tex]{\sf Z}[/tex] from the bearing of [tex]{\sf X}[/tex] from [tex]{\sf Z}[/tex] to find the measure of the angle [tex]\angle {\sf YZX}[/tex]:

[tex]\begin{aligned}\angle {\sf YZX} &= 135^{\circ} - 45^{\circ} \\ &= 90^{\circ}\end{aligned}[/tex].

Thus, triangle [tex]\triangle {\sf YZX}[/tex] is a right triangle ([tex]90^{\circ}[/tex]) with segment [tex]{\sf YX}[/tex] as the hypotenuse. It is given that [tex]|{\sf XZ}| = 6\; {\rm cm}[/tex] whereas [tex]|{\sf ZY}| = 6\; {\rm cm}[/tex]. Thus, by Pythagorean's Theorem:

[tex]\begin{aligned}|{\sf ZY}| &= \sqrt{|{\sf ZX}|^{2} + |{\sf ZY}|^{2}} \\ &= \sqrt{(8\; {\rm cm})^{2} + (6\; {\rm cm})^{2}} \\ &= 10\; {\rm cm}\end{aligned}[/tex].

Katrina drinks 0.5 gallons of water per day. Which expression shows how to find the number of cups of water she drinks in a week?

There are 16 cups in a gallon.

Answers

Answer:

D

Hope this helps!

Click an item in the list or group of pictures at the bottom of the problem and, holding the button down, drag it into the correct position in the answer box. Release your mouse button when the item is place. If you change your mind, drag the item to the trashcan. Click the trashcan to clear all your answers.
One leg of a right triangle is 2 inches and the hypotenuse is 6 inches.

Find the area of the triangle.

_[tex]\sqrt{x} \\[/tex]

Answers

We can see that the area of the triangle is: 5.65in².

What is a triangle?

A triangle is actually known to be a shape that has three sides, three angles and three vertices. It's also known as a plane shape.

In order to find the area of the triangle, we will use Pythagorean Theorem: c² = a² + b².

Where c = hypotenuse = 6in.

b = 2in.

6² = a² + 2²

36 = a² + 4

a² = 36 - 4 = 32

a = √32

Area of the triangle = ½ × base × height.

Where base = √32

height = 2

Thus: A = ½ × √32 × 2

A = √32 = 5.65in².

Learn more about area of triangle on https://brainly.com/question/17335144

#SPJ1

Easy Problem for you guys to solve!

The measure of an angle is 10 more than two-thirds of that of its complement. Find the angle and its complement.

Answers

The angle is 42 degrees and its complement is 48 degrees
The answer is 42 and 48

(6 + 7) + 8 = 21

Choose an equivalent addition sentence that shows the associative property of addition.

Answers

Answer:

6 + (7 + 8) = 21

Step-by-step explanation:

the associative property of addition is: (a + b) + c = a + (b + c)

So the equivalent addition sentence would be:

(6 + 7) + 8 = 6 + (7 + 8) as they both equal 21

Given the following data points, calculate the curve of best fit. show all steps.

Answers

Based on the calculations, the equation for the curve of best fit is equal to y = -30.17x + 14.49.

How to calculate the curve of best fit?

From the table of data points, we have the following:

∑x = 16∑y = 50.9∑xy = 24.6∑x² = 35

Mathematically, the standard equation of a straight line is given by:

y = ax + b       ....equation 1.

Thus, the equations that can be used to model the given data points are:

∑y = na + b∑x             ....equation 2.

∑xy = a∑x + b∑x²       ....equation 3.

Substituting the parameters into the equations, we have;

50.9 = 6a + 16b             ....equation 4.

24.6 = 16a + 35b       ....equation 5.

Solving eqn. 5 and 6 simultaneously, we have:

a = -30.17.b = 14.49.

Substituting the value of a and b into eqn. 1, we have;

y = ax + b

y = -30.17x + 14.49.

Therefore, the equation for the curve of best fit is equal to y = -30.17x + 14.49.

Read more on curve of best fit here: brainly.com/question/9366563

#SPJ1

Other Questions
Which is a reason why historians might change their interpretation of an event?A new piece of evidence is discovered.New technology allows faster publication of results.They may dislike the previously accepted interpretation.An old theory may be unpopular with the general public. Explain how promoting products derived from sustainable sources is intended to conserve rainforests. Which expression is to equivalent to.... P()P(A)0.40.1P(B)0.20.3 Consider the table below.x y-1 -50 51 112 133 11Complete the standard form equation representing the quadratic relationship displayed above, where a, b, and c are constants. What is the word equation for atoms of magnesium reacting with molecules of oxygen to produce magnesium oxide? For any real number c, =A. B. clC. 1D. C Question 8 of 10What must a speaker keep in mind to best appeal to the audience?A. The length of time the audience has been waiting to hear thespeakerB. Famous speeches the audience may know by heartC. The right of the audience to ask questionsOD. The expectations and knowledge of the audienceSUBMIT On Sunday, a local hamburger shop sold a combined total of 512 hamburgers and cheeseburgers. The number of cheeseburgers sold was three times the number of hamburgers sold. How many hamburgers were sold on Sunday? (1) Deep into that darkness peering, long I stood there wondering, fearing,(2) Doubting, dreaming dreams no mortals ever dared to dream before;(3) But the silence was unbroken, and the stillness gave no token,(4) And the only word there spoken was the whispered word, "Lenore?""The Raven,"Edgar Allan PoeWhich sound device is used with the words peering and fearing?Which sound devices are used in line 2?What mood do the sound devices create? What is the main purpose of the European Union? Show work, can someone help me out having trouble with this problem Sulfuric acid is an important industrial chemical thatis usually produced by a series of reactions. One ofthese involves an equilibrium between gaseous sulfurdioxide, oxygen, and sulfur trioxide.2 SO(g) + O(g) 2 SO(g)If 2.5 mol of sulfur dioxide gas and 2.0 mol ofoxygen gas are placed in a sealed 1.0 L containerand allowed to reach equilibrium, 0.75 mol of sulfurdioxide remains. Use an ICE table to determine theconcentration of the other gases at equilibrium. Why would an author choose to use satire?A. To present the facts and evidence that provide support for the author's main argumentB. To give readers information and clues about what will happen in the later part of the storyC. To make readers with opposing viewpoints consider an issue more deeplyD. To make a direct comparison between two things that do not appear to be outwardly alike Explain how light brings about tropic movement in the stem and root of aflowering plant. The gratuity on a restaurant bill was $18. If a 20% tip was left, what was the original amount of the bill? which choices are equivalent to the expression 3 square root of 5 Which of the following is the disadvantage of sexual reproduction?A. Formation of new characteristicsB. Increased diversity of the speciesC. More survival ratesD. Mate selection Page 1:the poet like an acrobat climbs on rime to a high wire of his own making-Constantly Risking Absurdity (#15),Lawrence FerlinghettiPage 2:rime (noun) an accumulation of ice or frostrime (noun) rhyming verseRead the excerpt from the poem and the two definitions. What is the effect of the poets use of the word "rime?It makes writing poetry seem challenging, like trying to climb something slippery. It makes writing poetry seem easy, like sliding up a smooth surface.It creates a metaphor that compares reading poetry to a circus performance.It creates a symbol that connects writing to nature. In a three-paragraph essay, describe an alternate treaty to end World War I that you feel would have been more successful than the Treaty of Versailles. As you describe your treaty, be sure to answer the following questions:What provisions would it include?How would these differ from the ones in the Treaty of Versailles?Why would your treaty be more successful in preventing postwar economic downturns or military conflicts?